Demostrar que para cualquier secuencia dada de dígitos, existe un cuadrado perfecto que comienza con esa secuencia

Demuestre que para cualquier secuencia dada de dígitos, existe un cuadrado perfecto que comienza con esa secuencia. Con más detalles, demuestre que para a norte , tal que a = a 1 a 2 . . . a norte ¯ , a i { 0 , . . , 9 } , i = 1.. norte ¯ y a 1 > 0 , allá pag norte tal que pag 2 = a 1 a 2 . . . a norte . . . ¯

Para resumir, en 1995 asistía a una escuela de verano de matemáticas y este problema era una "tarea" relacionada con el teorema de aproximación de Dirichlet y el teorema de aproximación de Kronecker , por lo que las soluciones proporcionadas deberían incluir estas herramientas (un poco de restricción al problema ).

La distancia entre 2 números que se saltan ese dígito inicial en particular es exponencial en términos de número de dígitos. La distancia entre cuadrados consecutivos es exponencial con base más baja en términos de número de dígitos.

Respuestas (2)

Probablemente esto no sea lo que desea, pero suponga que desea un cuadrado perfecto que comience con a 1 a 2 a norte entonces para suficientemente grande 2 k tendremos

10 k a 1 a 2 a norte + 1 10 k a 1 a 2 a norte = 10 k ( a 1 a 2 a norte + 1 a 1 a 2 a norte ) > 1 .

Esto significa que hay un cuadrado perfecto entre 10 2 k ( a 1 a 2 a norte ) y 10 2 k ( a 1 a 2 a norte + 1 ) Este número es un cuadrado perfecto que comienza con a 1 a 2 a norte

¿Por qué significa esto que hay un cuadrado perfecto entre esos números?
@DavidButlerUofA: Porque si X y y son números reales con X y > 1 , hay un número entero entre ellos. Aquí X = 10 k a 1 a 2 a norte + 1 y y = 10 k a 1 a 2 a norte .
Ah, claro. Dado que hay un número entero m entre x e y (es decir, X > metro > y ), hay un cuadrado de un número entero entre X 2 y y 2 (es decir X 2 > metro 2 > y 2 ).

Probablemente no esté de acuerdo con las reglas, pero después de unos días llegué a esta solución, que probablemente no sea la más elegante. Sigo buscando uno mejor...

  1. Si a es un cuadrado perfecto en sí mismo, hemos terminado. Ahora, supongamos...

  2. a no es un cuadrado perfecto, entonces a R q y tambien 1 a R q . Entonces, de acuerdo con el teorema de aproximación de Kronecker, METRO = { metro a + norte metro , norte Z } es denso en R . Esto básicamente significa que para X , y R , X < y allá γ METRO tal que X < γ < y .

si tomamos X = 10 k y y = 10 k + 1 a entonces metro , norte Z tal que

10 k a < norte a + metro < 10 k a + 1
notando
norte a + metro = α = α + { α }
Tenemos
10 k a 10 k a + 1 { α } < α + 1 < 10 k a + 1 + 1 { α } 10 k a + 2
Dónde α + 1 = pag estamos buscando. Entonces
10 k a < pag < 10 k a + 2
o
10 2 k a < pag 2 < 10 2 k a + 4 10 k a + 4
y obviamente ahí t norte tal que 10 2 k a + t = pag 2 . Esto impone la restricción
0 < t < 4 10 k a + 4
Lo que queremos ahora, teniendo en cuenta la naturaleza arbitraria de k , es
0 < t < 4 10 k a + 4 < 10 2 k
es decir
4 a + 4 10 k < 10 k
lo cual es cierto para k > registro 10 ( 4 a + 1 ) . De esta manera, tenemos
pag 2 = 10 2 k a + t , 0 < t < 10 2 k

¿Por qué molestarse con todo esto, cuando Bananarama ya publicó una prueba de cinco líneas?
Desde la perspectiva educativa “por lo que las soluciones aportadas deberían incluir estas herramientas (un poco de coacción al problema)”. ... Estoy seguro de que hay otras formas de demostrarlo.